LSAT and Law School Admissions Forum

Get expert LSAT preparation and law school admissions advice from PowerScore Test Preparation.

 Administrator
PowerScore Staff
  • PowerScore Staff
  • Posts: 8916
  • Joined: Feb 02, 2011
|
#25680
Complete Question Explanation

Strengthen—PR, CE. The correct answer choice is (C)

The doctor’s argument begins with the conclusion, that “it is wrong for medical researchers to keep their research confidential.” As support for this conclusion, the doctor points out that when researchers do not share their research, it is possible that effective medical treatments will be delayed, and that humans may suffer unnecessarily as a result.

To conclude that keeping research confidential is wrong, the doctor must be applying some normative rule. The stimulus does not give us the rule being applied, and so the argument is flawed.

The question stem identifies this as a Strengthen—Principle question, which asks us to select the answer choice containing the rule applied by the doctor to reach the conclusion. The doctor disapproves of keeping research confidential because doing so could delay effective medical treatments and possibly cause unnecessary human suffering. So, our prephrase, the rule being applied by the doctor, could be stated this way: an act is wrong if it could delay effective medical treatments and cause unnecessary human suffering.

Before we move to the answer choices, you may be wondering why we have not done anything with the second half of the first sentence, “even if the companies for which they work would rather that they do so.” This comment was merely an aside by the doctor, addressing the pressures that induce medical researchers to keep their research confidential. It does not tell us why the doctor thinks the act of not sharing medical research is wrong. So, for our prephrase, we need only take note of the aside, and be on guard to avoid an answer choice that tries to lure us in that direction.

Answer choice (A): Although this answer choice provides a rule, it is a rule that controls a different set of circumstances than what we saw in the stimulus. The doctor’s concern was the possibility of human suffering. This rule would apply if the researchers knew, for certain, that their actions would result in human suffering.

Answer choice (B): To reach the conclusion that it is “wrong” for medical researchers to keep their research confidential, we need a rule that applies to the facts provided in the stimulus. The stimulus does not tell us that preventing human suffering is the most important moral principal, so this answer choice has no impact on the conclusion.

Answer choice (C): This is the correct answer choice, because it provides explicitly the rule used by the doctor to reach the conclusion. We know from the stimulus that there is the chance that keeping the research confidential would cause unnecessary human suffering. So, while it is not certain, there is the chance that sharing the research could prevent unnecessary human suffering. Therefore, this conditional rule applies to the stimulus, requiring that medical researchers not keep their research confidential.

Answer choice (D): This answer choice is irrelevant, because it deals with developing research instead of disclosing research results.

Answer choice (E): This choice is irrelevant too, because the conclusion had to do with the researchers’ moral obligations, rather than the companies’ obligations.
 netherlands
  • Posts: 136
  • Joined: Apr 17, 2013
|
#9389
Hey there PS,

I got this one wrong. I think that I might have been scrutinizing details in the answer choices incorrectly. I'm looking back at them and trying to understand why C was the best answer choice. Can you let me know whether these methods/understanding of elimination are right?

A) This choice says that researchers shouldn't engage in any behavior that researchers "know" will cause humans to suffer. In the stimulus though the doctor states that treatment "may" be delayed... in which case it seems that he's saying information shouldn't be held regardless of whether potential suffering can be known, but simply because it's "likely"/"possible".

B) "If the most important moral principle is to..." - I just really didn't feel like this qualifier is what the doctor is claiming in the stimulus. He doesn't say anything about preventing human suffering being the most important moral principle.

- I'm not sure why I crossed off C, maybe rushing. But now after looking at why A and B seem wrong and D and E are irrelevant I think I may see why C was the correct coice.

Can you confirm or correct?

Thanks!
 Steve Stein
PowerScore Staff
  • PowerScore Staff
  • Posts: 1153
  • Joined: Apr 11, 2011
|
#9392
Hey Netherlands,

This is a Strengthen question, so we need to look at the author's argument and find the principle that most helps.

The Doctor says that if research is not shared, humans may suffer unnecessarily.
The Doctor's conclusion is that it is wrong for researchers to keep their work confidential.

The correct answer choice will be the one that supports the doctor's conclusion that the potential for human suffering makes it wrong for researchers to keep their work confidential.

Answer choice C is the only choice that fits perfectly the strengthen, or "most help justify" the doctor's argument.

I hope that's helpful! Let me know--thanks!

~Steve
 netherlands
  • Posts: 136
  • Joined: Apr 17, 2013
|
#9393
Ok well, A could also strengthen it - so what differentiates the two?
 Steve Stein
PowerScore Staff
  • PowerScore Staff
  • Posts: 1153
  • Joined: Apr 11, 2011
|
#9394
Hey Netherlands,

Thanks for your response. The difference there comes down to some subtle wording. The correct answer choice C deals with situations in which it is possible that human suffering could be prevented. This is consistent with the soft language from the stimulus, in which the doctor provides only that humans may suffer unnneccessarily. Answer choice A, in contrast, deals with situations in which researchers know that certain behavior will cause human suffering. Since this is not necessarily relevant to the doctor's discussion, which deals only with potential suffering, it is does not strengthen the doctor's argument.

I hope that's helpful! Please let me know whether this is clear--thanks!

~Steve
 netherlands
  • Posts: 136
  • Joined: Apr 17, 2013
|
#9395
Hey there,

So does that mean that what I wrote out in my first post for the elimination of answer choice "A" was correct? You didn't confirm or deny in your first response and that's pretty much what I was originally asking you to comment on - were my methods of elimination correct.

It's super helpful when I'm having trouble with an answer to know whether or not the reason I eliminate answer choices is correct. ;)

Thanks!
 Steve Stein
PowerScore Staff
  • PowerScore Staff
  • Posts: 1153
  • Joined: Apr 11, 2011
|
#9396
Thanks for your response,

Regarding what you were originally asking, A was properly ruled out, and when you wrote "I think I may see why C is the correct answer choice," that sounded a lot like a request for clarification as to why that was the case. ;)

~Steve
 netherlands
  • Posts: 136
  • Joined: Apr 17, 2013
|
#9397
Ok sounds good. Your letting me know that it was in fact correct to pay attention to those subtle wordings confirmed what I was thinking! There are some points where I'll see the subtle differences, but second-guess and worry that I may be just finding reasons to eliminate under pressure.


Thank you!
 lsat2016
  • Posts: 59
  • Joined: May 29, 2016
|
#25898
Hello,

I eliminated E on the basis that:

premise: not share -> delay -> suffer unnecessarily

but E: delay-> wrong

however, C: suffer unnecessarily -> wrong

since E does not connect suffer unnecessarily -> wrong, it is incorrect?

I chose C because it connects the last element (suffer unnecessarily) to the conclusion.

Is this a valid way of eliminating an answer choice??
 Nikki Siclunov
PowerScore Staff
  • PowerScore Staff
  • Posts: 1362
  • Joined: Aug 02, 2011
|
#25915
Hi lsat2016,

While I see how you approached the question, the logically valid reasons for eliminating each of these answer choices are outlined above. I feel like you're buried too deep into the conditional constructions to understand the argument properly and evaluate it holistically. :) Check out this blog post:

Conditional Reasoning: Do You See It Everywhere?

Thanks,

Get the most out of your LSAT Prep Plus subscription.

Analyze and track your performance with our Testing and Analytics Package.